2023年6月4日 星期日

112年台北市國中聯合教甄-數學詳解

臺北市112 學年度市立國民中學正式教師聯合甄選

貳、專業科目
選擇題(共 40 題,每題 1.75 分,共 70 分)

解答:$$\begin{vmatrix}8 & a \\b & 9 \end{vmatrix}=72-ab=1 \Rightarrow ab=71為質數 \Rightarrow (a,b)=(71,1)或(1,71)\\ \Rightarrow a+b=72, 故選\bbox[red,2pt]{(D)}$$
解答:$$(a+bi)(2+6i)=2a-6b+(6a+2b)i =50 \Rightarrow \cases{2a-6b=50\\ 6a+2b=0} \Rightarrow \cases{a=5/2\\ b=-15/2} \\ \Rightarrow a+b=-5, 故選\bbox[red,2pt]{(A)}$$
解答:$$x=4有最大值\Rightarrow \cases{p\lt 0\\ q=4}故選\bbox[red,2pt]{(D)}$$
解答:$$假設\cases{甲獨做需a天\\ 乙獨做需b天\\ 丙獨做需c天 } \Rightarrow \cases{1/a+ 1/b=1/12 \cdots(1)\\ 1/b+1/c=1/15 \cdots(2)\\ 1/c+1/a=1/20 \cdots(3)}\\ 將(2)及(3)代入(1) \Rightarrow {1\over 15}-{1\over c}+{1\over 20}-{1\over c}={1\over 12}  \Rightarrow {2\over c}={2\over 60} \Rightarrow c=60,故選\bbox[red,2pt]{(D)}$$
解答:$$\left( 2x^2-{1\over x}\right)^9 =\sum_{n=0}^9 C^9_n(2x^2)^n (-{1\over x})^{9-n} \Rightarrow x^6的係數=C^9_5\cdot 2^5 (-1)^{4} =4032, 故選\bbox[red,2pt]{(D)}$$


$$假設長方形\cases{長邊長=2a\\ 短邊長=2b}\Rightarrow 面積=4ab=4 \Rightarrow ab=1\\ 箏形ABCD面積=ab=1\Rightarrow 覆蓋總面積=3個長方形-3個箏形=3\cdot 4-3\cdot 1=9, 故選\bbox[red,2pt]{(B)}$$
解答:

$$梯形如上圖, 面積= 6+6=12, 故選\bbox[red,2pt]{(B)}$$
解答:

$$等角六邊形,每一內角為120^\circ,因此延長\overline{AB}, \overline{CD}, \overline{EF}可得正\triangle PQR, \\且\triangle PAF,\triangle BQC,\triangle FDR皆為正\triangle ,如上圖\\ 假設\cases{\overline{PA}= \overline{PF}=a \\ \overline{RD}= \overline{RE}=b\\ \overline{EF}=c}, \overline{PQ}=\overline{QR}=\overline{PR} \Rightarrow 19+a=17+b \Rightarrow b=a+2且c=17-a\\ 周長=51=a+10+9+8+b+c= 27+19+a \Rightarrow a=5 \Rightarrow c=17-a=12, 故選\bbox[red,2pt]{(B)}$$
解答:$$(1)\times: {1\over 9}=0.1111\cdots為有理數\\ (2)\times: \pi為無理數,\pi^n 仍為無理數,n\in \mathbb N \\(4)\times: \sqrt 2+(-\sqrt 2)=0為有理數\\ (5)\times: \sqrt 2\times \sqrt 2=2為有理數\\ 只有(3)是正確的, 故選\bbox[red,2pt]{(A)}$$
解答:$$\cases{x\ge 1/2 \Rightarrow x^2-2x-3=0 \Rightarrow (x-3)(x+1)=0 \Rightarrow x=3\\ x\lt 1/2 \Rightarrow x^2+2x-5=0 \Rightarrow x=-1-\sqrt 6}\\ \Rightarrow 所有解的和=3-1-\sqrt 6=2-\sqrt 6, 故選\bbox[red,2pt]{(B)}$$
解答:$$\overline{AB}的中點(2,5,13/2)投影至xy平面為(2,5,0),故選\bbox[red,2pt]{(C)}$$
解答:$$取\cases{n=7a\\ m=7b},其中a\gt b \Rightarrow n^2+m^2=49(a^2+b^2) = 1666 \Rightarrow a^2+b^2=34 \\ \Rightarrow \cases{a=5\\ b=3} \Rightarrow n-m= 5\cdot 7-3\cdot 7=14, 故選\bbox[red,2pt]{(B)}$$
解答:$$2007^{2007}= (286\cdot 7+5)^{2007} =\sum_{n=0}^{2007} C^{2007}_n(286\cdot 7)^n\cdot 5^{2007-n}\\ 上式除了n=0外,其他皆是7的倍數,因此只要考慮n=0的項次,即5^{2007} \\由於\cases{5^0 \equiv 1 \mod 7 \\5^1 \equiv 5 \mod 7 \\5^2 \equiv 4 \mod 7 \\5^3 \equiv 6 \mod 7 \\5^4 \equiv 2 \mod 7 \\ 5^5 \equiv 3 \mod 7 \\5^6 \equiv 1 \mod 7 } \Rightarrow 循環數=6 \Rightarrow 2007=6\times 334+3 \Rightarrow 餘數=6, 故選\bbox[red,2pt]{(D)}$$
解答:$$題目有誤,題目應該是f(x)={2x-\color{blue}{3c}\over (a-b)(a-c)}+{(2x-2c)(x-3c)\over (b-c)(b-a)} +{(x-3c)(2x-3c)\over (c-a)(c-b)}\\ 令a=2b=3c=6k \Rightarrow \cases{a=6k\\ b=3k \\ c=2k}\\ \Rightarrow f(a+b+c)=f(11k) ={16k\cdot 9k\over 3k\cdot 4k} +{90k^2\over -3k^2}+{80k^2\over 4k^2} =12-30+20=2,故選\bbox[red,2pt]{(C)}$$
解答:$$9\times c的個位數字為3\Rightarrow c=7 \Rightarrow (9b+6+21)的個位數字為4 \Rightarrow b=3\\ \Rightarrow 原式變為6537\times a39=6537\times 39+ 6537\times 100a=254943+653700a=156de43 \\ \Rightarrow 6537a+2549=156de \Rightarrow \cases{a=2\\d=2\\ e=3} \Rightarrow 239\times 6537=1562343\\ \Rightarrow \cases{a=2\\ b=3\\ c=7\\ d=2\\ e=3},故選\bbox[red,2pt]{(C)}$$
解答:$$-2\lt x\lt 4 \Rightarrow (x-4)(x+2)\lt 0 \Rightarrow f(x)=-k(x-4)(x+2), k\gt 0\\ 因此 f(3x)\lt 0 \Rightarrow -k(3x-4)(3x+2)\lt 0 \Rightarrow (3x-4)(3x+2)\gt 0 \Rightarrow x\gt {4\over 3}或x\lt -{2\over 3},故選\bbox[red,2pt]{(B)}$$
解答:$$a=\sqrt{8+\sqrt{45}} =\sqrt{8+3\sqrt{5}} \Rightarrow a^2=8+3\sqrt 5 \approx 14.XXX (\sqrt 5\approx2.YYY) \\ \Rightarrow 9\lt a^2\lt 16\Rightarrow 3\lt a\lt 4, 故選\bbox[red,2pt]{(D)}$$
解答:$$f(x)=\sum_{k=1}^{10} (1-x)^k \Rightarrow f'(x)= \sum_{k=1}^{10} -k(1-x)^{k-1} \Rightarrow f''(x)=\sum_{k=2}^{10} k(k-1)(1-x)^{k-2} \\ f(x)的x^2的係數={1\over 2}f''(0) ={1\over 2}\sum_{k=2}^{10} k(k-1) ={1\over 2}(385-55)= 165,故選\bbox[red,2pt]{(D)}$$
解答:$$\begin{array}{rr|r} a& b& 數量\\\hline 15& -15至15& 31\\ -15 & -15 至15 & 31\\ -15至15 & 15 & 31\\ -15至15 & -15 &31\end{array} \Rightarrow 合計=31\times 4=124,扣除重複計算的(\pm 15,\pm 15)\\ 因此格子點共有124-4=120個, 故選\bbox[red,2pt]{(C)}$$

解答:

$$\cases{\overline{AB'}=3\\ \angle B'AB=30^\circ} \Rightarrow \cases{\overline{B'R}= 3/2\\ \overline{AR}=3\sqrt 3/2} \Rightarrow \overline{B'Q}=2-\overline{B'R}={1\over 2}\\ 又\angle QB'C'= 30^\circ  \Rightarrow \overline{QS}=1/2\sqrt 3 \Rightarrow \cases{梯形B'PCS面積={3\over 2}-{17\over 24}\sqrt 3\\ 梯形ABPB'面積={9\over 2}-{9\over 8}\sqrt 3} \\ \Rightarrow 兩梯形面積=6-{11\over 6}\sqrt 3 \Rightarrow 重疊面積={11\over 6}\sqrt 3,故選\bbox[red,2pt]{(B)}$$

解答:

$$正六角形內角=(6-2)\times 180^\circ\div 6=120^\circ \Rightarrow \cos \angle PAB=\cos 120^\circ  \\\Rightarrow \triangle PAB面積={1\over 2}\cdot 1\cdot {1\over 2}\sin 120^\circ ={\sqrt 3\over 8},灰色面積=4\times {\sqrt 3\over 8}={\sqrt 3\over 2}, 故選\bbox[red,2pt]{(A)}$$
解答:
$$直角\triangle BFE \Rightarrow \overline{BE}=\sqrt{6^2+8^2}=10\\ 令\angle BEF=\theta \Rightarrow \sin\theta={3\over 5} \Rightarrow \cos \angle BED=\cos(90^\circ+\theta)=-\sin \theta=-{3\over 5}\\ 又\cos \angle BED={10^2+10^2-\overline{BD}^2\over 2\cdot 10\cdot 10}=-{3\over 5} \Rightarrow \overline{BD}=\sqrt{320} \Rightarrow \overline{AD}=\sqrt{160} \\ \Rightarrow 正方形面積=\overline{AD}^2=160, 故選\bbox[red,2pt]{(D)}$$
解答:$$(x^2-x-1)^{x+10}=1 \Rightarrow \cases{x+10=0 \Rightarrow x=-10\\ x^2-x-1=1 \Rightarrow x=2,-1\\ x^2-x-1=-1且x+10為偶數\Rightarrow x=0} \\ \Rightarrow x=-10,-1,0,2共四個整數解,故選\bbox[red,2pt]{(C)}$$
解答:$$2017-\sqrt x=0^2,1^2,2^2,\dots,44^2(45^2=2025) \Rightarrow x有45個整數解,故選\bbox[red,2pt]{(B)}$$
解答:$${(2^2-1)(3^2-1)(4^2-1)\cdots(100^2-1)\over 1^2\times 2^2\times 3^2\times \cdots \times 100^2} ={(1\times 3)(2\times 4)(3\times 5)\cdots (99\times 101)\over 100!\times 100!} \\={99!\times 101!/2\over 100!\times 100!} ={1\over 100}\times {101\over 2} \approx {1\over 2}=0.5, 故選\bbox[red,2pt]{(B)}$$
解答:$$三內角的和=180^\circ 為偶數,而\cases{偶數=奇數+奇數+偶數\\ 偶數=偶數+ 偶數+偶數};\\ 由於偶質數只有一個(2),因此180=奇數(a)+奇數(b)+2 \Rightarrow a+b=178\\ \Rightarrow \begin{array}{rr|r}a& b& \\\hline 1& 177 & 177不是質數\\ 3& 175 &175不是質數\\ 5& 173 & \bigcirc\\ 7& 171& 171不是質數\\ 11& 167& \bigcirc\\ 13& 165 &\times\\ 17& 161& 161=7\cdot 23\\ 19& 159 & \times\\ 23& 155 & \times\\ 29& 149 & \bigcirc\\ 31 & 147& \times\\ 37& 141& \times \\ 41& 137 & \bigcirc\\ 43 & 135& \times\\ 47& 131& \bigcirc\\ 53& 125 & \times\\ 59& 119&119=7\cdot 17\\ 61 & 117&\times\\ 67 & 111 & \times \\ 71 & 107& \bigcirc\\ 73 & 105 &\times\\ 79& 99&\times \\ 83 & 95& \times\\ 89 &89 & \bigcirc \end{array} \Rightarrow 共7個, 故選\bbox[red,2pt]{(B)},但公布的答案是\bbox[blue,2pt]{(C)}$$
解答:$$a^2-6a-1997=0 \Rightarrow\cases{a^2=6a+1997 \Rightarrow a=6+{1997\over a}\\ a=3+\sqrt{2006}}\\ \Rightarrow 8+{1997\over 6+{1997\over 6+{1997\over 6+{1997\over a}}}} =  8+{1997\over 6+{1997\over 6+{1997\over a}}} = 8+{1997\over 6+{1997\over a}} = 8+{1997\over a} \\=2+a =5+\sqrt{2006}, 故選\bbox[red,2pt]{(A)}$$
解答:


$$三頂點為直角各有兩個C點,共有3\times 2=6個C點,如上圖,故選\bbox[red,2pt]{(D)}$$
解答:$$\cases{a-b=(\sqrt{13}-\sqrt{12})-(\sqrt{15}-\sqrt{16}) =正-負=正\gt 0\\ b-c=(\sqrt{12}-\sqrt{11})-(\sqrt{16}-\sqrt{17})=正-負=正 \gt 0} \\ \Rightarrow \cases{a\gt b\\ b\gt c} \Rightarrow a\gt b\gt c,故選\bbox[red,2pt]{(A)}$$
解答:$$|3-||x+2|-2|=5 \Rightarrow \cases{3-||x+2|-2|=5 \Rightarrow ||x+2|-2|= -2(不合,絕對值需\ge 0) \\3-||x+2|-2|=-5 \Rightarrow ||x+2|-2|=8} \\ \Rightarrow \cases{|x+2|-2=8 \Rightarrow |x+2|=10\\ |x+2|-2=-8 \Rightarrow |x+2|=-6(不合,絕對值需\ge 0)} \Rightarrow x=8,-12, 故選\bbox[red,2pt]{(B)}$$
解答:$$\cases{\log_5 a=11\\ \log_5 b=13} \Rightarrow \cases{a=5^{11} \\ b=5^{13}} \Rightarrow \log_5(a+b)= \log_5(5^{11}+ 5^{13}) =\log_5 5^{11}(1+5^2) \\ =11+\log_5(1+5^2)=11+2.XX =13.XX, 故選\bbox[red,2pt]{(B)}$$
解答:$$0.33+0.0303+ 0.003003+ 0.00030003+\cdots\\ =(0.3+0.03)+ (0.03+ 0.0003)+ (0.003+0.000003)+ (0.0003+ 0.0000003)+ \cdots\\ = \sum_{k=0}^\infty (0.3\cdot 0.1^k +0.03\cdot 0.01^k) =0.3\cdot {10\over 9}+ 0.003\cdot {100\over 99}={12\over 33}={4\over 11}, 故選\bbox[red,2pt]{(B)}$$
解答:$$二次方程式通過原點,因此有一直線過(0,0)及(1,-2),該直線為2x+y=0\\ \Rightarrow 2x^2+axy+3y^2+bx+cy=(2x+y)(x+3y+k) \Rightarrow 兩直線為\cases{L_1:2x+y=0\\ L_2:x+3y+k=0} \\ \Rightarrow \cases{\vec u=(2,1)\\ \vec v=(1,3)} \Rightarrow \cos \theta ={\vec u\cdot \vec v\over |\vec u||\vec v|}={5\over \sqrt{5}\cdot \sqrt{10}}={\sqrt 2\over 2} \Rightarrow \theta={\pi\over 4},故選\bbox[red,2pt]{(B)}$$
解答:


$$\cases{\angle A=30^\circ\\ \angle B=45^\circ} \Rightarrow \angle C=105^\circ,在\overline{AB}上找一點P,使得\angle PCB=45^\circ \Rightarrow \angle PCA=60^\circ,如上圖\\ 假設\overline{PC}=\overline{PB}=a \Rightarrow \cases{\overline{AP}=\sqrt 3a\\ \overline{AC}=2a\\ \overline{BC}=\sqrt 2a} \Rightarrow \triangle ABC周長=(3+\sqrt 2+\sqrt 3)a=3+\sqrt 2+\sqrt 3\\ \Rightarrow a=1 \Rightarrow \overline{BC}=\sqrt 2,故\bbox[red,2pt]{(無解)},公布的答案是\bbox[blue, 2pt]{(D)}$$
解答:$$x^2+4y^2=1 \Rightarrow x^2=1-4y^2 \Rightarrow 3x^2+2y=3-12y^2+2y\\ =-12(y^2-{1\over 6}y+{1\over 144})+3+{1\over 12} =-12(y-{1\over 12})^2+{37\over 12} \Rightarrow 最大值={37\over 12}, 故選\bbox[red,2pt]{(D)}$$
解答:


$$立方體展開圖有11種,其中T型(如上圖),有最大矩形空間161,故選\bbox[red,2pt]{(C)}$$
解答:$$\cases{0^2+2^2=4 \equiv 4 \mod 8\\ 1^2+3^2=10 \equiv 2 \mod 8\\ 2^2+3^2=13 \equiv 5\mod 8}, 故選\bbox[red,2pt]{(D)}$$

解答:$$  {\overline{AD}\over \overline{DE}}={\triangle ADF\over \triangle DEG} ={1\over 1} \Rightarrow 可假設\overline{AD}= \overline{DE}=k \Rightarrow \overline{AE}=2k\\ {\overline{AE}(2k)\over \overline{EF}} ={\triangle AEH\over \triangle EFH}={3\over 1} \Rightarrow \overline{EF}={2\over 3}k \Rightarrow \overline{AF}={8\over 3}k\\ 又{\overline{AF}\over \overline{FB}} ={\triangle ACF\over \triangle BCF}={5\over 1} \Rightarrow \overline{FB}={8\over 15}k \Rightarrow {\overline{AD} \over \overline{FB}}={k\over 8k/15}={15\over 8},故選\bbox[red,2pt]{(B)}$$
解答:


$$令\cases{A(0,0)\\ B(10,0)} \Rightarrow C(10,5) \Rightarrow L=\overleftrightarrow{AC}: 2y=x \Rightarrow \cases{P=(x=2)\cap L =(2,1)\\ Q=(x=5)\cap L=(5,5/2)}\\ \Rightarrow \cases{\overline{PS}=3-1=2\\ \overline{QR}=3-5/2=1/2} \Rightarrow 梯形PQRS面積={(2+1/2)\cdot 3\over 2}={15\over 4}=3.75, 故選\bbox[red,2pt]{(C)}$$

解答:$$最上層只有一個骰子,有五個面,最小的五面數字和=1+2+3+4+5=15\\ 第二層有四個骰子,最小的四面字和=(1+2+3+4)\times 4=40\\ 底層有8個骰子,其中4個僅有一面,此四面數字和=4;另四個骰子顯示三面,數字和=(1+2+3)\times 4=24\\ 因此最小的數字和=15+40+4+24=83,故選\bbox[red,2pt]{(D)}$$


================= END ==============

解題僅供參考,其他教甄試題及詳解

2 則留言:

  1. 老師請問66題是7組嗎? 因為a+b=178,當a=83時,b應該是95,這組應該不是

    回覆刪除
    回覆
    1. 對耶!, 應該是7組,不曉得有沒有試題疑義更正!

      刪除